segunda-feira, 28 de abril de 2008

Fantástico o que Celina Bruniera escreve aqui :
Vale a pena ler .

"Ao lermos um texto escrito, o sucesso de nossa leitura se dá na medida em que procuramos estabelecer uma interação com o autor. Se considerarmos que o leitor é um agente, que não tem um papel passivo durante o ato de ler, mas que é um sujeito que atribui significado ao texto, que procura as pistas deixadas pelo autor, que busca se aproximar do sentido que este quer dar ao texto, podemos fazer da leitura um lugar onde há reciprocidade.
Em relação aos textos do gênero argumentativo (artigos, editoriais, cartas de reclamação, etc.), textos em que se procura convencer o leitor a aceitar uma opinião ou mesmo adotá-la para si, desvendar as intenções do autor materializadas em argumentos que sustentam uma posição e em evidências que, por sua vez, ancoram os argumentos, significa estar aberto a rever suas próprias opiniões.

Nossas pré-noções
Quando lemos, nem sempre estamos atentos às pistas textuais, aquelas dicas deixadas pelo autor para que nos aproximemos do que este quis nos dizer. Muitas vezes, isso ocorre porque - ao usarmos todo o nosso conhecimento prévio (lingüístico, textual e de mundo) na ação de ler - o sentido que atribuímos ao texto é mediado por nossas pré-noções.
Aquilo que pensamos sobre o assunto em questão, nosso conhecimento do gênero textual e da língua nos conduzem a elaborar algumas hipóteses sobre o texto. No entanto, um leitor cuidadoso e interessado deve rever a pertinência de suas hipóteses ao longo da atividade de leitura e buscar se aproximar da intenção comunicativa.
Alguns elementos do texto podem contribuir muito para que isso seja possível, sobretudo quando se trata da leitura de um texto em inglês ou em outra língua estrangeira. Isso porque como o texto não é escrito na nossa língua e, portanto, não dominamos muito bem as características do contexto sociocultural em que se deu a situação de produção discursiva, encontraremos mais dificuldade em nos aproximarmos de suas intenções. Para esse esforço ser minimizado, vale a pena conhecer como o texto foi elaborado.

A elaboração do texto
O texto objeto de nossa análise é uma resenha de um livro supostamente de interesse de áreas como economia, ciências humanas, meio ambiente e planejamento. Uma leitura rápida dos elementos destacados no texto nos fornece essas informações. Basta ver o rodapé em que essas áreas e o preço do livro são revelados, e o que está escrito entre parênteses.
Em geral, as resenhas de livros veiculadas nas capas dos mesmos têm como intenção convencer o leitor de que sua leitura vale a pena. Além disso, procuram apresentar o tema tratado na publicação e o enfoque dado a ele. Assim, apresentam um viés argumentativo muito explícito, tanto no sentido de convencer o leitor a ler o livro como no sentido de promover sua adesão ao enfoque dado ao tema.
A informação trazida entre parênteses e o título da resenha nos sugerem que a resenha aborda o tema da fome. Antes de ler, tente imaginar como esse tema será apresentado no texto e, ao ler, procure checar suas hipóteses.

Why are so many hungry?
A Pelican Original
“If it takes you six hours to read this book, somewhere in the world 2, 500 people will have died of starvation or of hunger-related illness by the time you finish.
Why are so many hungry ? Susan George affirms with conviction, and with solid evidence, that it is not because there are too many passengers on ‘Spaceship Earth’, not because of bad weather or changing climates, but because food is controlled by the rich. Only the poor go hungry.
The multinational agribusiness corporations, Western governments with their food ‘aid’ polices and supposedly neutral multilateral development organizations share responsibility for their fate. They all work in cooperation with local elites, themselves nurtured and protected by the powerful in the developed world. United States agripower paves the way, leads the pack and is gradually imposing its control over the whole planet.
Only those fortunate people who can become consumers will eat in the Brave New World being shaped by the well-fed. The standard liberal solutions to feeding the world -- population control or the Green Revolution -- are just what the hungry poor don’t need. All they need is social change, otherwise known as justice. With that, they could, and would, resolve most of their problems themselves.’
Cover design by John Carrod, photograph by Rod Stone
Economics
Science
Environment and Planning
United Kingdom £ 1,00
Canada $ 2,50
(back cover of How the other half dies by Susan George)

Para nos aproximarmos das intenções do texto, um dos elementos que merecem destaque é o uso do léxico, mais precisamente a escolha das palavras, dos substantivos, verbos, adjetivos e advérbios.
Ao apresentar a tese da autora sobre as razões que levam as pessoas a morrerem de fome (a comida é controlada pelos ricos), notamos a força expressiva de certos advérbios, tais como many ("hungry") e only ("the poor"). Ainda nesse mesmo parágrafo, é fácil perceber que a opinião da autora é ressaltada por meio do verbo to affirm e do substantivo conviction ("Susan George affirms with conviction"), além do adjetivo solid ("evidence").
Nesse segundo parágrafo, é possível notar que o uso dos conectivos because (sobretudo em "because de food is controlled by the rich") e but garantem, respectivamente, a explicitação do que leva as pessoas à fome e a oposição da tese da autora em relação a outras teses veiculadas sobre o mesmo assunto.

Para refletir e motivar
Antes desse parágrafo, porém, o texto nos oferece um parágrafo introdutório no qual a relação estabelecida entre o tempo médio despendido para a leitura do livro e o número de pessoas que morrem de fome ou em virtude de doenças relativas a ela sugere a reflexão acerca do tema e motiva o interesse pela leitura da resenha e, conseqüentemente, do livro.
No terceiro parágrafo, o texto levanta uma série de evidências que, apesar de generalizantes, dão força à tese defendida e ampliam nosso conhecimento sobre o tema. No caso, vale a pena realizar uma análise do voaculário utilizado e dos elementos mobilizados pelo texto para ancorar a tese defendida. Tente fazer o movimento que fizemos em relação ao segundo parágrafo e identificar com mais propriedade a intenção comunicativa do texto.

* Celina Bruniera é mestre em Sociologia da Educação pela USP e assessora educacional para a área de linguagem.
http://educacao.uol.com.br/ingles/ult1703u28.jhtm

A TESE DO AUTOR E SEUS ARGUMENTOS :

Em
As intenções do texto(1) já iniciamos a análise da resenha "Why are so many hungry?". Essa análise tem buscado evidenciar as pistas deixadas pelo autor para materializar sua intenção comunicativa. Reproduzimos a resenha novamente.

Why are so many hungry?
A Pelican Original
“If it takes you six hours to read this book, somewhere in the world 2, 500 people will have died of starvation or of hunger-related illness by the time you finish.
Why are so many hungry ? Susan George affirms with conviction, and with solid evidence, that it is not because there are too many passengers on ‘Spaceship Earth’, not because of bad weather or changing climates, but because food is controlled by the rich. Only the poor go hungry.
The multinational agribusiness corporations, Western governments with their food ‘aid’ polices and supposedly neutral multilateral development organizations share responsibility for their fate. They all work in cooperation with local elites, themselves nurtured and protected by the powerful in the developed world. United States agripower paves the way, leads the pack and is gradually imposing its control over the whole planet.
Only those fortunate people who can become consumers will eat in the Brave New World being shaped by the well-fed. The standard liberal solutions to feeding the world -- population control or the Green Revolution -- are just what the hungry poor don’t need. All they need is social change, otherwise known as justice. With that, they could, and would, resolve most of their problems themselves.’
Cover design by John Carrod, photograph by Rod Stone
Economics
Science
Environment and Planning
United Kingdom £ 1,00
Canada $ 2,50
(back cover of How the other half dies by Susan George)

Já havíamos analisado os dois primeiros parágrafos do texto e deixamos como atividade que você analisasse os dois parágrafos finais. E isso é o que faremos aqui.

Ressaltar evidências
Já dissemos que a tese apresentada por Susan George, autora da obra "How the other half dies" e reproduzida na resenha, é explicitada no segundo parágrafo. Susan George afirma que há muitas pessoas com fome porque a comida é controlada pelos ricos. Tendo deixado clara a posição a ser defendida, o texto caminha no sentido de ressaltar os argumentos e evidências que lhe dêem sustentação.

O terceiro parágrafo apresenta as corporações multinacionais de agronegócios, as políticas de "ajuda" dos governos ocidentais e as organizações de desenvolvimento multilateral como responsáveis pelo destino dos pobres, únicos que passam fome. Observe que o uso do advérbio supposedly (supostamente) e do adjetivo neutral (neutras) evidencia uma opinião acerca das organizações de desenvolvimento multilateral.

Ainda nesse parágrafo, vale notar que o adjetivo powerful ganha status de substantivo quando o texto afirma que os responsáveis pelo destino dos pobres trabalham em cooperação com as elites locais e que estas são alimentadas e protegidas pelos poderosos (powerful) no mundo desenvolvido. Aqui o texto revela um recurso discursivo bastante comum, isto é, o uso de nomes (substantivos) para se referir a alguém ou alguma coisa e, dessa forma, revelar como se quer qualificar esse alguém ou essa coisa.

Um último aspecto a ser ressaltado no terceiro parágrafo diz respeito ao uso dos verbos to pavê (pavimentar), to lead (liderar) e to impose (impor) para se referir ao poder exercido pelo setor agrícola norte-americano em relação ao controle da comida ao redor do mundo.

Mundo dos bem-alimentados
No último parágrafo, a resenha apresenta as conclusões a que chega Susan George em seu livro "How the other half dies". A primeira conclusão é a de que terão comida apenas aqueles que puderem se tornar consumidores num mundo desenhado pelos bem-alimentados. Essas pessoas que terão acesso à comida são qualificadas como fortunate people (pessoas com sorte) e viverão no "Admirável Mundo Novo", uma alusão à obra de Aldous Huxley ("Brave New World"), em que as relações sociais se mostram pautadas na completa separação entre os ricos e os pobres e a vida dos ricos é marcada pelos avanços tecnológicos e pela racionalização de todas as esferas da vida. A alusão a essa obra consiste em mais um elemento significativo para nos aproximarmos das intenções do texto.
A segunda conclusão apresentada pela resenha da obra é a de que a Green Revolution (uma revolução na agricultura) não resolveria o problema dos pobres famintos (observe a adjetivação the hungry poor). Essa revolução é caracterizada no texto como uma das típicas soluções liberais para alimentar o mundo (standart liberal solutions to feeding the world). A última conclusão a que chega Susan George é de que para resolver o problema daqueles que têm fome é preciso mudança social (social change), também conhecida como justiça (justice).

http://educacao.uol.com.br/ingles/ult1703u28.jhtm

Gírias / Expressões

MARKETING E VENDAS MARKETING

Correspondência comercial. - Business writing. / Business letters.
Os clientes não estão fazendo muitos pedidos. - The customers are not placing many orders.
O vendedor está atendendo um cliente. - The salesman is helping a customer.
Propaganda é a alma do negócio. - It's all marketing. / It pays to advertise.
O cliente vem sempre em primeiro lugar. / O cliente sempre tem razão. - The customer is always right.
Encontrar um denominador comum. - Find common ground.
Está à venda. / Vende-se. - It's up for sale. / For sale.
Em liquidação. / Em promoção. - On sale. / Clearance.
Remarcado em 20% - 20% off
Novinho em folha - Brand new.
Fora de linha - Discontinued.
É uma barbada. / É uma pechincha. - It's a good deal. / It's a real bargain.
Fiz uma boa compra. - I got a good deal.
É um roubo. - It's a rip-off.
Fui roubado. - I got ripped off.
Cheque sem fundo. - Bad check. / Bounced check. / Rubber check.
Cheque pré-datado. - Post-dated check.
Condições de pagamento - Terms of payment.
A prazo / Em prestações / No crediário - In installments.
De entrada / Como sinal - As a down payment.
O restante / O saldo - The remaining balance / The balance.
Pagar à vista, em dinheiro. - Pay cash.
Pagar adiantado. - Pay in advance.
No atacado / A preços de atacado - At wholesale. / At wholesale prices.
No varejo / A preços de varejo - At retail / At retail prices.
__________________________________________

NO TRABALHO AT WORK
Normalmente vou a pé para o trabalho, mas quando chove vou de carro. - I usually walk to work, but when it rains I drive. / ... I take my car.
Ele ganha 1.000 dólares por mês. - He makes a thousand dollars a month.>>>>
Hoje é dia de pagamento. - Today's payday.>>>>
A secretária está atendendo o telefone. - The secretary is answering the phone. / ... is on the phone.>>>>
Favor informar - Please let me know>>>>
Você pode deixar um recado na secretária eletrônica. - You can leave a message on the answering machine.>>>>
Não vou poder assistir à reunião hoje de tarde. - I won't be able to attend the meeting this afternoon. / I'm not going to be able ... / I'm not able ... / I can't ...
Proibida a entrada de pessoas estranhas ao serviço. - Personnel only. / Unauthorized entry prohibited.
O horário de trabalho (expediente) é das 8 às 12. - Working hours are from>> 8 to 12.>>>> Após o horário de expediente ? - After working hours. / After hours.>>>>
Durante o horário comercial. - During business hours.>>>>
Tenho que fazer hora extra. - I have to work overtime.>>>>
O horário de verão nos EUA vai de abril a outubro. - Daylight saving time>> in the US is from April to October.>>>>
Faltam dois dias para eu entrar em férias. - I'll go on vacation>> (holidays) in two days. / There are two days left before I go on vacation.>>>>
Está faltando alguém? - Is anybody missing?>>>>
Está faltando dinheiro no mercado. - There is a shortage of money in the market.>>>>
Faz dois anos que eu trabalho aqui. - I've been working here for two years.>>>>
Eu trabalhava num banco, antes. - I used to work for a bank.>>>>
Fiquei sabendo que ele foi demitido. / Ouvi dizer que ... - I heard he was>> fired. / ... he was dismissed. / I was told that he was ...>>>>
Um novo gerente será contratado. - A new manager will be hired.>>>>
Quem manda aqui sou eu! - I'm the boss around here!>>>>
Preencha a ficha (formulário) de inscrição. - Fill out the application>> form.>>>>
Ele está de plantão. / Ele está de serviço. - He's on call. / He's on duty.>>>>
Ele está aqui a serviço. / ... a negócios. - He's here on business.>>>>
Vou tirar uma folga amanhã. - I'm going to take a day off.>>>>
Ele vai se aposentar. - He's going to retire.>>>>
Ela está de licença. - She's on leave.>>>>
Ela está encostada no INPS. / ... de licença para tratamento de saúde. ->> She's on sick leave.>>>>
O sindicato não está cooperando. - The (labor) union is not cooperating.>>>>
Os trabalhadores estão planejando fazer greve. - The workers are planning to go on strike.>>>> A/C (aos cuidados de). - C/O (care of).>>>>
Já foi providenciado. - It's been taken care of.>>>>
Todos os funcionários devem bater o cartão-ponto. - All the workers must punch their time cards.
__________________________________________
Anniversary and birthday
O substantivo anniversary não pode ser empregado como "aniversário de nascimento". É usado para referir-se a "aniversário de casamento"; "aniversário de fundação de uma empresa ou instituição"; ou "aniversário de uma cidade". Veja os exemplos:
The Dawsons are giving a big party to celebrate their wedding anniversary.Os Dawson vão dar uma grande festa para comemorar o aniversário de casamento deles.
Our company is celebrating its fiftieth anniversary next week.Nossa empresa vai comemorar o 50º aniversário de fundação na próxima semana.Então, como dizer aniversário de nascimento? Birthday.
Did you get many presents for your birthday?Você ganhou muitos presentes de aniversário?

Audience
Esse substantivo é comumente empregado para referir-se a "público ou platéia de show ou concerto". Pode ser também traduzido por "audiência" no sentido de "reunião formal com alguém muito importante", como o papa, o presidente ou um rei.
The audience cheered as the rock band came on stage.A platéia vibrou quando a banda de rock subiu ao palco.

Como dizer "audiência"?Para se referir a "sessão de tribunal em que testemunhas são ouvidas", podemos empregar os termos court hearing ou court session:
"Since one of the witnesses was not present, a new court hearing was scheduled for next Friday", explained the lawyer."Como uma das testemunhas não estava presente, uma nova audiência foi marcada para a próxima sexta", explicou o advogado.Observação: No sentido de "ibope de programa de TV ou rádio", ratings é a palavra apropriada:
The TV network executives are not at all satisfied with the ratings the new talk show has been getting.Os executivos da rede de TV não estão nada satisfeitos com a audiência que o novo talk show está obtendo.


Babaca: asshole
Neste caso, você tem várias opções. Além de asshole, pode usar dickhead, jackass e jerk I, cujos significados são bem próximos ao de "babaca".
Jay is such an asshole! I wonder why you hang around with him all the time.Jay é tão babaca! Não sei por que você está sempre com ele.


Bajular: to butter up
Eis aí algo que muita gente gosta de fazer, em geral para conseguir vantagens...
Shirley knows how to butter people up whenever she wants a favor from them.Shirley sabe como bajular as pessoas sempre que quer um favor delas.


Bater o telefone na cara de alguém: to hang up on someone
É, às vezes não dá para agüentar. Basta uma explosão de raiva e pronto: batemos o telefone, cortando a conversa. E deixando, do outro lado da linha, alguém perplexo ou furioso. Veja como dizer isso em inglês:
Sandy can be really rude when she wants to. Can you believe she hung up on me last night?Sandy sabe mesmo ser mal-educada quando quer. Você acredita que ela bateu o telefone na minha cara ontem à noite?


Beef
As aparências realmente enganam! O substantivo beef não pode ser traduzido por "bife". Ele designa, isto sim, a "carne bovina":
Nancy is a vegetarian. She never eats beef or pork.Nancy é vegetariana. Ela nunca come carne de vaca nem de porco.
Como dizer "bife"?
A palavra steak significa "bife". Observe, contudo, que steak normalmente se refere a "fatia grossa de carne", não necessariamente de vaca:
"How would you like your steak, sir? Rare, medium or well-done?", the waiter asked Walter."Como o senhor gostaria do seu bife? Mal passado, no ponto ou bem passado?", o garçom perguntou a Walter

City and town
O substantivo city é usado para cidades grandes e importantes:
"I sometimes wish I could move away from this crazy city to a calmer town in the countryside", Mike told his friends."Às vezes eu tenho vontade de me mudar desta cidade louca para uma cidadezinha mais calma no interior", Mike disse aos amigos.O substantivo town é normalmente usado para cidades pequenas, maiores que uma village (aldeia, povoado), mas menores que uma city:
Greg had a hard time getting used to living in New York since he grew up in a small town.Foi difícil para Greg acostumar-se a morar em Nova York, já que ele cresceu numa cidade pequena.

Cobaia: guinea pig
Se você ainda não assistiu ao filme Laranja mecânica, assista. É o lado tenebroso das cobaias - e dos contextos também tenebrosos que podem levar a elas. Mas nosso exemplo é mais simples. Vamos falar da ciência, que, ao menos teoricamente, pauta-se pela ética.They are asking for volunteers to be guinea pigs in their new medical research.Estão pedindo voluntários para serem cobaias na nova pesquisa médica.


Da hora :

AWESOME[impressionante; legal, da hora, animal]
Your new car is really awesome!
O teu carro novo é mesmo da hora!

Mala e cuia

Roberto saiu daqui de mala e cuia dizendo que não volta mais.
Karina mudou-se de mala e cuia para o interior paulista.
A gente chegou aqui de mala e cuia em 1985.
Tô de mala e cuia pro Rio de Janeiro
"De mala e cuia" neste contexto quer dizer que a pessoa simplesmente juntou tudo o que tem e esta partindo ou chegando a algum lugar. É usada para dizer que vamos ficar em definitivo no novo local ou que estamos de férias apenas, porém, levamos tudo o que era necessário para ficar bem.Além de "mala e cuia" é possivel ainda ouvir alguém dizer "as traia" (as tralhas), dito assim mesmo com artigo no plural e o substantivo no singular. Claro que "as traia" prode anda se referir a muitas outras coisas.Como dizer "de mala e cuia" em inglês? Perguntou um leitor do blog! Enfim, como dizer que alguém mudou-se, partiu, chegou, saiu, está de mala e cuia para algum lugar?Em inglês, há a expressão "bag and baggage". Teve sua origem no contexto militar. Ou seja, quando um grupo de soldados deixa um local limpinho, nada ficando para trás eles diziam "retreat bag and baggage" (bater em retirada de mala e cuia). Assim, a expressão saiu do contexto militar e ganhou popularidade entre os civis. Veja alguns exemplos:
When they went to collect the rent, they found he had left, bag and baggage. (Quando foram cobrar o aluguel, descobriram que ele tinha partido de mala e cuia.)
The missionary was going, bag and baggage, to another village. (O missionário estava indo de mala e cuia para outra vila.)
They arrived, bag and baggage, to their new home last night. (Eles chegaram de mala e cuia na casa nova ontem a noite.)
We arrived here, bag and baggage, in 1992. (A gente chegou aqui de mala e cuia em 1992).
Fonte:Denilso de lima


Diretoria
board of directors/boardFique esperto na hora da entrevista para o emprego: "diretoria" não é directory, é board! Observe o exemplo.
The board will meet today to discuss a new strategy to boost sales.A diretoria vai se reunir hoje para discutir uma nova estratégia para aumentar as vendas.

Discussão acirrada
heated argumentNem pense em discussion! Discussão é argument. E, quando acirrada, é heated argument.
They had a heated argument about who should pay for the damages after the car crash.Tiveram uma discussão acirrada sobre quem deveria pagar pelo prejuízo após a batida.

Distraído
absent-minded Distraído, desligado, desatento, alheado... Quem vive com a cabeça nas nuvens é, decididamente, absent-minded.
I wonder how Jerry can be so absent-minded. He keeps leaving his cell phone behind wherever he goes. Não sei como o Jerry consegue ser tão distraído. Ele está sempre esquecendo o celular em todos os lugares a que vai.

Dizer o que se pensa
to speak one's mind-Há um velho ditado para aqueles que costumam dizer o que pensam: "Quem diz o que quer, ouve o que não quer". Em outras palavras, é o que o exemplo recomenda:
You can't always speak your mind. Sometimes you have to keep your opinions to yourself.Não se pode sempre dizer o que pensa. Às vezes, é preciso guardar para si as opiniões pessoais.

Dormir como uma pedra
to sleep like a logEis outra "pegadinha": nesta expressão, pedra não é stone. É log.
Mike was so tired that he slept like a log last night.Mike estava tão cansado que dormiu como uma pedra ontem à noite.

Chega! É a gota d'água!
Se estivermos muito bravos e dissermos, a um inglês, That's enough! That's a water's drop! (Chega! É a gota d'água!), ele provavelmente não irá entender. Mas, se falarmos That's enough! That's the last straw! , ele saberá que chegamos a uma situação-limite, daquelas em que não é mais possível tolerar algum acontecimento ou fato desagradável que vem se repetindo.Como você notou, a expressão coloquial equivalente à nossa "gota d´água" é, em inglês, the last straw (a última palha). Observe que a expressão completa "A gota d'água que faz o balde transbordar" tem sua equivalência em inglês em The last straw that breaks the camel's back (A última palha que quebra as costas do camelo).Veja o exemplo contextualizado:
When Fred showed up late for work for the third time in a row, his boss told him that it was the last straw and that he would have to fire him.Quando Fred chegou atrasado ao trabalho pela terceira vez seguida, seu chefe lhe disse que aquilo era a gota d'água e que ele teria de despedi-lo.

Leão-de-chácara
Bouncer
Many nightclubs have a bouncer to throw out troublemakers.Muitas boates têm leão-de-chácara para expulsar encrenqueiros.Observação: o substantivo bouncer, formado do verbo to bounce (quicar, fazer saltar, pular), é usado para referir-se àquela figura, normalmente musculosa, presente em boates e danceterias, e sempre disposta a colocar para fora os encrenqueiros (troublemakers) e outras pessoas indesejáveis, fazendo-os quicar.


Mala e cuia
Roberto saiu daqui de mala e cuia dizendo que não volta mais. Karina mudou-se de
mala e cuia para o interior paulista. A gente chegou aqui de mala e cuia em 1985. Tô de mala e cuia pro Rio de Janeiro "De mala e cuia" neste contexto quer dizer que a pessoa simplesmente juntou tudo o que tem e esta partindo ou chegando a algum lugar. É usada para dizer que vamos ficar em definitivo no novo local ou que estamos de férias apenas, porém, levamos tudo o que era necessário para ficar bem.Além de "mala e cuia" é possivel ainda ouvir alguém dizer "as traia" (as tralhas), dito assim mesmo com artigo no plural e o substantivo no singular. Claro que "as traia" prode anda se referir a muitas outras coisas.Como dizer "de mala e cuia" em inglês? Perguntou um leitor do blog! Enfim, como dizer que alguém mudou-se, partiu, chegou, saiu, está de mala e cuia para algum lugar?Em inglês, há a expressão "bag and baggage". Teve sua origem no contexto militar. Ou seja, quando um grupo de soldados deixa um local limpinho, nada ficando para trás eles diziam "retreat bag and baggage" (bater em retirada de mala e cuia). Assim, a expressão saiu do contexto militar e ganhou popularidade entre os civis. Veja alguns exemplos: When they went to collect the rent, they found he had left, bag and baggage. (Quando foram cobrar o aluguel, descobriram que ele tinha partido de mala e cuia.) The missionary was going, bag and baggage, to another village. (O missionário estava indo de mala e cuia para outra vila.) They arrived, bag and baggage, to their new home last night. (Eles chegaram de mala e cuia na casa nova ontem a noite.) We arrived here, bag and baggage, in 1992. (A gente chegou aqui de mala e cuia em 1992). Fonte:Denilso de lima

Matriz/sede/escritório centralHeadquarters
Our company's headquarters is in Miami.A matriz da nossa empresa é em Miami.Observações: no contexto militar, headquarters (comumente abreviado para HQ) significa também "quartel-general".O termo head Office também designa o escritório principal de uma empresa:
Where is the head office of your company located?Onde se localiza o escritório central de sua empresa?


Manda-chuva/figurão/pistolão/bambambãBig shot
Who's the big shot around here?Quem é o manda-chuva por aqui?Outro termo informal, bigwig, também é usado nesse contexto:
Mr. Wayne is a bigwig in the world of advertising.O sr. Wayne é um figurão no mundo da propaganda.


Mexer os pauzinhosTo pull strings
Do you think you can pull some strings to speed up the shipment of our products?Você acha que pode mexer os pauzinhos para acelerar a expedição de nossos produtos?

MalpassadoRare
Para dizer "bem-passado" e "no ponto", usa-se respectivamente os termos well-done e medium.
Waiter: Would you like to order now, sir?
Tom: Yeah. I'd like a green salad and a medium steak, please.Garçom: O senhor gostaria de fazer o pedido agora?Tom: Sim. Eu queria uma salada verde e um filé no ponto, por favor.


Marriage and wedding
Em português, casamento pode designar tanto a instituição, quanto a cerimônia, certo? Em inglês, há duas palavras para isso.Com o sentido de "instituição do casamento", diz-se marriage.
Peter and Maggie's marriage lasted only two years, since Peter was too jealous and Maggie decided to break up.O casamento de Peter e Maggie durou apenas dois anos, já que Peter era ciumento demais e Maggie resolveu separar-se.Com o sentido de "cerimônia de casamento", diz-se wedding.
Over one hundred people attended Tom and Liz's wedding.Mais de cem pessoas foram ao casamento de Tom e Liz.

Na hora H


You arrived just in the nick of time.
Você chegou justo na hora H.

My car was in excellent nick, but somebody nicked it last week. Yesterday the police nicked the thief and now he’s in the nick, but my car’s in really bad nick.
Meu carro estava em excelente estado, mas alguém o roubou na semana passada. Ontem a polícia prendeu o ladrão, e agora ele está na prisão, mas o meu carro está em péssimo estado.

No fundoDeep down/deep inside
Deep down Jeff knew that what he was doing was wrong. [Ou: Deep inside Jeff knew that what he was doing was wrong.]No fundo, Jeff sabia que o que ele estava fazendo era errado.Observaçãodeep down e deep inside referem-se a sentimentos. "Fundo", na acepção oposta à de "superfície", é bottom.
"Voyage to the Bottom of the Sea" was a well-known TV series in the early seventies."Viagem ao Fundo do Mar" era um seriado famoso no começo da década de 70.



Quase nunca
Once in a blue moon
As Burt's in-laws live very far away, he only visits them once in a blue moon.Como a família da esposa de Burt mora muito longe, ele quase nunca os visita.Observação:Os advérbios very seldom e hardly ever, menos informais, também são muito usados nesse contexto.

Rachar a conta
To go dutch
"Let's go dutch!", Susan told Bob when the waiter brought them the check."Vamos rachar!", Susan disse a Bob quando o garçom trouxe a conta.Observações:1. A expressão to go dutch normalmente se restringe ao sentido de "rachar contas de refeições".2. Outra expressão alternativa, esta mais genérica, é to split the Bill.


Saideira / bater as botas / bater as botas / pior das hipóteses
Suponhamos, agora, que você esteja num bar londrino, consulte o relógio e veja que é hora de ir para casa. Como faria para convidar os amigos a um último drinque, a nossa famosa "saideira"? Bem, nesse caso a expressão equivalente é one for the road .
"Let's have one for the road!", Jack told his friends at the pub. "Vamos tomar a saideira!", Jack disse a seus amigos no pub .Terminada a saideira, você sai do bar e, ao cruzar com um casal punk, ouve a seguinte frase: "Did you know that old Mr. Spencer kicked the bucket?" Claro que você não vai concluir, apressadamente, que o velho sr. Spencer "chutou o balde" - a tradução literal de kicked the bucket . Como já vimos aqui, o significado dessa expressão é "bater as botas", morrer. Por isso, você saberá a tradução correta:
"Did you know that old Mr. Spencer kicked the bucket?" "Você soube que o velho sr. Spencer bateu as botas?"A caminho do hotel, você percebe que pegou o ônibus errado. Mas não se preocupa muito. Na pior das hipóteses, vai conhecer um percurso novo. Além disso, basta permanecer naquele mesmo ônibus para voltar à rua do bar e pegar a condução certa. Só uma coisa deixa você intrigado: como falar "na pior das hipóteses" em inglês? Anote: the worst-case scenario . Veja o exemplo:
What's the worst-case scenario if everything else goes wrong? Qual é a pior das hipóteses caso tudo mais dê errado?


Sei lá!Beats me!
Barry: - What does the acronym LASER mean? Jay: - Beats me! Try looking it up in a dictionary.Barry: - O que a abreviação LASER significa?Jay: - Sei lá! Tente procurar num dicionário.Observações:1. LASER: Light Amplification Stimulated by the Emission of Radiation. 2. Search me! é outra expressão informal com o mesmo significado de Beats me!.

Terceirização
Outsourcing
Outsourcing of non-core activities is a usual practice among many companies nowadays.A terceirização das atividades não-principais é prática comum em muitas empresas hoje em dia.Observação:Para traduzir o verbo "terceirizar", use to outsource:
Many companies prefer to concentrate on their core business and outsource other departments.Muitas empresas preferem concentrar-se em suas atividades principais e terceirizar outros departamentos.

“Tesão/ter tesão”: to be horny; to have a hard on (só homens); to be randy (usada na Inglaterra, mas me parece um pouco antiquada); to have wood (só homens); to have a woody (só homens); to be hot; to have the hots for (someone); to want it; to feel like it; I’m wet (for you) (só mulheres).
“Você não presta”: You’re no good; You’re worthless; You’re nothing.


Torcer (1)
To root for a teamCom o sentido de "torcer para um time":
Gary started rooting for the L.A. Lakers ever since he moved to Los Angeles.Gary começou a torcer pelo Lakers depois que se mudou para Los Angeles.

Torcer (2)To keep one's fingers crossedCom o sentido de "torcer para que algo dê certo ou aconteça do jeito que se quer; fazer figa":
I'm keeping my fingers crossed that all your projects will work out fine.Estou torcendo para que todos os teus projetos dêem certo.Observação: To root for também se usa informalmente na acepção "torcer para alguém":
You can do it! I'm rooting for you!Você consegue! Estou torcendo para você!

Torcer (3)To wringCom o sentido de "torcer roupa, pano etc.".
Make sure you wring the clothes before you hang them out on the line. Não deixe de torcer as roupas antes de pendurá-las no varal.

Torcer (4)To twistCom o sentido de "torcer o pulso, o tornozelo, o joelho etc.".
Martha twisted her wrist playing volleyball.Martha torceu o pulso jogando vôlei.Observação: O verbo to sprain também é bastante usado no mesmo contexto:
Tom slipped on a banana skin and sprained his ankle.Tom escorregou numa casca de banana e torceu o tornozelo.

Uma questão de
A matter of
"Don't worry! Learning how to cook well is only a matter of time", Mrs. Smith told her teenage daughter."Não se preocupe! Aprender a cozinhar bem é só questão de tempo", a sra. Smith disse à filha adolescente.Observação:A expressão question of também é usada com o mesmo significado de a matter of.

FONTE : José Roberto A. Igreja é diretor pedagógico da Dialecto English, e autor dos livros "How do you say ... in English ?" e "Falsos Cognatos - Looks can be deceiving!" (Disal Editora).
http://educacao.uol.com.br/ingles/

domingo, 27 de abril de 2008

TER = haver

Fonte : Denilson de Lima http://denilsodelima.blogspot.com


In Portuguese it's very common to say sentences like the following (como as seguintes):
Tem mais de 50 pessoas na minha sala.
Tem um cara aí fora te esperando.
Tem uns livros sobre a cama. Pode tirar!

The thing is (O lance é o seguinte): we, Brazilian Speakers of Portuguese, in our everyday conversation, prefer using "tem" to "há" in sentences like these. "Há" is very formal, "tem" is more colloquial!That's where the problem starts! (É aí que o problema começa!) O problema está justamente no fato de muitos brasileiros ao falar inglês traduzem este "tem" por "have". Enquanto o correto é usar "there is" ou "there are". Qual a diferença?"There is" é sempre usado com uma coisa só, por exemplo:

There is a guy waiting for you outside. (Tem um cara esperando por você aí fora)
There is a carton of milk in the fridge. (Tem uma caixinha de leite na geladeira)Já o "there are" é usado para mais de uma coisa:
There are more than 50 people in my classroom. (Tem mais de 50 pessoas na minha sala.)
There are some books on the bed. You can take them outta there! (Tem uns livros em cima da cama. Você pode tirá-los de lá!)
Por mais simples que pareça este assunto e por mais básico que seja, muita gente hora ou outra acaba falando "have a boy in the room" (tem um menino no quarto). Portanto, lembre-se neste contexto o certo é usar "there is" ou "there are".Comece a fazer algumas sentenças para você ir se acostumando com isto. Um dica é começar a escrever coisas que "tem" (há) no seu escritório, quarto, casa, dentro do seu carro... Enfim, comece a usar o "there is" e o "there are" para não falar coisas sem sentido depois!That's it! Take care... See you on Monday!

sábado, 19 de abril de 2008

Kumon Questions and Answers

Olha o que eu encontrei na net ... muito legal !


These are emails I have received in response to my main Kumon page. I've responded as honestly as possible. There's no useful order; you have to skim to see which exchanges may be of interest to you.

***
THEE: I am looking for some tutoring for my daughter not because she is
bad in any particular subject but because she is playful and I want
something to help her concentrate and help her organize her thoughts
while she writes. I was looking at Oxford learning centre and Kumon. Do
you know anything about Oxford method by any chance? They say that their
teacher to student ratio is 1 to 3 and they have personalized program
for each child based on his/her needs. I am not sure how true that is.
They get $150.00 for the assessment test (which take about 2 hours) and
$170.00 per month for 4 hours. Comparing the prices Kumon is way more
economical but they say that there is no teacher. Kumon says that
students get homework for 7 days on each visit which is 15 minutes and
they have to do it on their own. I am not sure if this is going to be
helpful for my child.
ME: Unfortunately, I know absolutely nothing about the Oxford learning
centre. If it sounds a little bit "to good to be true", I wouldn't
necessarily hold that against them; doesn't that apply to almost
everything that's marketed? My position on student/teacher ratio is
very radical - so reader beware! While we are supposed to believe that
the smaller the student/teacher ratio the better, I believe that there
is no significant difference between 3 students or 40 per teacher. I am
convinced the magic number is 2, which has many advantages over one-on-
one, even. Three becomes a class; and a teacher can't latch onto three
student brains at once. At that point, dialog becomes lecture - with
the attendant uncertainty of everything being "taught" being grasped by
all of those good, little, attentive(?) students.
You didn't say the age of your daughter. If her writing is your main
concern, I presume she's already a developed reader. If that's the
case, I don't think the Kumon reading program would have much to offer.
And more extensive writing doesn't come about until much later in the
Kumon program. And when the student does write more than a few words,
there's no one at Kumon who has the time or experience necessary to give
it more than a dry comparison with the workbook answer.
Having said all of that, if your local Kumon doesn't lock you into a
long term plan right from the start, it wouldn't hurt to give it a
chance. Maybe the regimen of doing Kumon worksheets 15 minutes a day is
just the thing for your daughter's "playfulness". (I hope they don't
kill it completely!) You'll have to be open-minded about the low
starting point, though.
***
THEE: I just read most of your comments in preparation for hosting an
exchange student who listed Kumon as an after school activity--I was
expecting a martial art, and it appears I was not that far wrong.
As you emphasize, the needs/capacities of the student and a tutor who
fits them are the ingredients for success. I hope parents follow your
suggestion of doing the worksheets--they will soon know if Kumon is a
good fit for their child.
Thanks offering your even-handed, revealing perspective..
ME: Thanks very much for the supportive words. I get occasional
feedback on my Kumon page, but very rarely any comment on the main point
- my entreaty to parents to do what the student has to do. If a program
is ineffectual, it will be found out. If a program is good, that's when
the magic starts.
***
THEE: i read your article on kumon. most parents think kumon is great,
but little do they know what the system is really like, much less the
curriculum. i have a relative who runs a kumon in california. i can't
believe so many people pay so much money for worksheets! if only
parents just took time to teach at home. kumon is a business. it comes
from an asian culture that prioritizes excellence in education at all
costs. that philosophy turns me off. excellence in character and
values and actions is what we need to strive for. kumon is not for
everyone. its methods are mind numbing. it does not inspire. i had
once thought of opening up a kumon myself. thought i might open one
with that aforementioned relative. in retrospect, i would have died
buried under the worksheets, bored and numbed by the mechanics. math is
wonderful. reading is wonderful. there is such a thing as the spirit
of the as well as the letter of the law. the spirit of math and reading
is lost with kumon. knowledge without spirit is no knowledge at all.

ME: Thanks for your thoughts. Of course, I am in almost complete
agreement. What little Kumon has going for it stems from how hopeless
our schools are (generally speaking). Kumon math may be murderous, but
at least *some* students may pick up *some* arithmetic skills from it.
The schools couldn't care less - just railroad a kid on to the next
level whether or not he's learned anything. I think Kumon's reading
program for the very beginning readers may be quite good; thereafter,
you could do a lot better with a stack of comic books or anything else
you can get a kid to read. And Kumon may be the only thing in a child's
life that requires a sort of work routine before getting out into the
work world. Too bad the material has no "spirit", as you put it.
***
THEE: I stumbled across your website while doing a search about Kumon
Learning Centers. I won't take up a lot of your time, I am sure you get
a lot of emails.
My question is this; I am going to be applying for a Manager Trainee
position at a local Kumon center here in Texas. What is a good math
review website or other source to prepare myself for the Math test that
Kumon requires? I would appreciate any thoughts you have on this.
I completed up to trigonometry in college but have not used it for
years....I have been a jazz pianist for the last 20. Thanks in
advance...
ME: "Stumbled"??? I'm Number 3, man!
I'm always glad to be of assistance, but I'm not sure I understand your
situation. I don't know what a "Manager Trainee" is; I only know of the
Instructor and the Assistants at a Kumon center. Does this mean a Kumon
franchisee can nowadays just sit back and hire a manager to run the
center???
Kumon is mostly a self-study program. Sometimes an instructor or
assistant might find a moment to help a struggling student, but it's not
a formal part of the program; not something the customer is paying for.
Kumon management would not even want a center to get heavily into that
because the center would then deviate from other centers. Since Kumon
is a franchise, that is not considered acceptable.
As far as math requirements are concerned, be assured that only a small
fraction of Kumon students ever gets beyond basic arithmetic, and a
tinier fraction yet gets beyond introductory algebra. The students that
get beyond that are whiz kids who won't need assistance, anyway.
***
THEE: Thanks for your insights on Kumon. I have a 5 year daughter who is
somewhat easily distractible with a short attention span. Do you think
that these worksheets and constant repetition would help her with math
and reading better? I am looking to give her more structure and
repetition.

ME: There are no guarantees in life, but it sounds like the Kumon
regimen very well might help your daughter. At her age, the worksheets
should be as much fun as work.
***
THEE: What is the cost to have a child tested at your facility? What is
the cost per hour for tutoring?

ME: For information on Kumon, go to their main site by typing "kumon" in
Google. They will say there is no cost to have your child tested, but
there is a registration fee of about $50 if you enroll. I see figures
given in news articles of about $35 - $70 dollars per hour as the
average for a private tutor. Of course, the sky's the limit. That may
sound expensive, but a half-hour session per week compares very
favorably with Kumon.
***
THEE: Thank you for your honest opinion of Kumon. I have been
considering sending my daughter to Kumon for a while.
My daughter is in 2nd grade. Her school gives an addition and
subtraction math test every month of 45 problems each. Each test has a
time limit of 1 min. Although, she tests at a 4th grade level, she only
gets about 15-17 of the problems done in 1 min. They are supposed to be
able to do the entire sheet of 45 problems before entering 3rd grade. It
seems like she slows down every time there is a time limit involved. Do
you think Kumon could help with this?
She has a grasp of numbers and can solve many word problems, so I know
that she has the skills. I also sit with her while she does her homework
and often do the problems in my head while looking over her shoulder.
Sometimes I time her without her knowing and she will do her math
homework (often word problems) in 5 to 10 mins depending on the number
of problems on the sheet. I am starting to wonder if she has some
psychological block with being timed.
ME: There are no garantees in life, but I think Kumon might help your
daughter get over the timing anxiety. After all, she'll be doing timed
assignments every day, so if that doesn't get her used to it, I can't
imagine what will. Of course, the thing isn't to terrorize a student
with a timer, and it isn't even the exact completion time or whether it
beats some arbitrarily set limit that matters. What you would be
looking for, and nudging her towards, is a state of always keeping the
pencil moving comfortably along. That's proof of a student knowing what
she's doing. She doesn't have to be a speed demon about it; if that's
what her school wants, they should be closed down.
I also need to ask, are you really sure she has the single digit
additions down pat? Does she have number sense ways of doing "hard"
additions like 7+9? Is she completely beyond counting, externally or
internally, in her additions? Does she have a crystal clear
understanding of subtraction being nothing new or separate, just
addition going the other way?
***
THEE: I was wondering if you could share your opinion with regards to
which private math programs in your opinion stand above the rest?

ME: I wish I could answer your question. It took a few years to get to
know Kumon, so you can see it's not feasible for me get familiar with
the other hundreds or thousands of programs people have come up with. I
still have no idea what goes on in a Sylvan or Huntington. I look at
things from the perspective of a competent private tutor. It absolutely
does not matter to me what math program a customer chooses. If it has
weaknesses I will see them and turn them into positives working with the
student. "See, you know better than the people who wrote this book!"
***
THEE: I am a current 8th grader who finished Kumon reading and is in
Level L in math. I cannot stand it anymore and my parents cannot help.
Do you have any advice for me? Thank you
Sincerely,
A Kumon disliker
ME: When you say your parents can't help, do you mean they won't listen
to your request to stop? There should be a law that a child can demand
that his parents do everything they make him do, but I guess that's a
way off yet. Have they read my thoughts on the lack of value of Kumon
above the arithmetic levels? I hope that will soften them. Best of
luck. You've certainly paid your dues.
By the way, did you get one thing out of Kumon's reading program that
you wouldn't get out of reading the things you ordinarily read and like
to read?
THEE: When I say my parents can't help, I mean that the stuff in Level L
is too complex for them. I don't have the time to stay in Kumon for an
hour or so either. My mom says that I have forgotten the basics for this
level (which is true) but every page is different. I don't get a chance
to actually learn the material and use it.
Kumon reading for me was useless. The only reason I passed was because
the first time that I did a worksheet, I would get everything wrong. But
then I would memorize the answer (It took some tries to get it 100%
correct but I managed). How do you feel about Kumon reading. I will
definetely show my mom your page. Thank you.
ME: In my Kumon web page I try to come across as a reasonable,
thoughtful, even-tempered sort of guy. So it's kind of hard to come out
and say, "Kumon's reading program, above the elementary levels, is a
miserable joke and a criminal waste of a child's life." $50 worth of
comic books would be more beneficial than $5000 worth of Kumon reading.
Good luck in getting out of Kumon and on with your life.
***
THEE: No words can describe how true the things you wrote about kumon
are. I am currently a high school freshman in level N of Kumon, and I
would have to agree that most of the practices that are fundamental to
the program are fundamentally WRONG. My Kumon center is stricter then
most, and I can't begin to describe the long hours that the program has
taken from my life. True, the idea is a good one, but improvements must
be made. As they say, the best is the enemy of the good, and there is
no reason to strive for perfection at the cost of lagging years behind
grade level. So, simply put, I would like to thank you for getting
these facts out there, and I hope that some people will take notice!

ME: Thanks for sharing something of your Kumon experience. At least it
sounds like you're holding up pretty well in spite of the drain on your
life. About my Kumon page, so far no one has ever written to say, "You
don't know what you're talking about!"
***
THEE: I found what you had to say about Kumon interesting. My son (6th
grade) is desperate to get out of Kumon jail. Can you suggest some
alternatives? He's on level E and does need the practice but really
dislikes Kumon and has since he started in 4th grade.

ME: I guess the main question is, why does your son hate doing the work?
Does he naturally hate doing any sort of academic work? That might
sound horrible, but it probably describes the majority of kids since the
beginning of time. I don't have any suggestions regarding kids who
won't work; I'll leave that problem to others.
Or is it that the work is frustratingly difficult? Students will say
they "hate" an assignment, or it's "boring", but the problem is really
that they don't have a complete understanding, and that makes the work
very painful. What I mean is, if a student has the multiplication table
down pat, doing a page of long divisions should be as satisfying (fun?)
as doing a word search, or crossword puzzle, or sudoku, etc. If that's
the problem, maybe irregular sessions with a private tutor would get
your son over Kumon's hurdles. No system is truly and completely "self-
study". Of course, if you went with a private tutor, you might as well
drop Kumon and find workbooks that aren't so dry and barren.
My last idea relates to my suggestion to all parents to do everything
the child does. Might it be more fun, and therefore more productive, if
he had a "teammate"?; if he didn't feel dumped into something on his
own? You might make a game of it, he does a problem that you check,
then you do a problem that he checks, etc. Kumon never has to know.
THEE: Thank you for responding; I truly appreciate it. I like your idea
of doing the homework along with my child and will give it a shot in
math and science to see if he becomes more engaged.

Do you know of any good/fun workbooks or CD's for 6th grade math?

ME: If working with your son on his homework as a friendly team - as
opposed to a master-pupil relationship - works, let the world know, or
let me know so I can pass it on to the world. I don't see how it can't,
except in the case of insurmountable personality clash problems. I met
an Indian a few weeks ago and he told me that when he was a child and
starting to take a "new math" course in India, a parent or relative
close to him had to practically sign a contract that he (the adult)
would learn the material along with the student. Sure makes sense to
me. This practice of just "dumping" kids into this, that, and the other
- something not a parent in a million even realizes he's doing - has to
stop.
Sorry for the rant.
About workbooks, I would just take the child to a bookstore and let him
pick out something that looks the most fun or interesting. I would
start with a 3rd-grade book. If he really is solid at that level, you
(plural) should have a blast spinning through it.
***
THEE: read your site on Kumon - very insightful. I just wanted to send
you an email & maybe pick your brain & learn from your experience w/ the
Kumon franchise. I was completely planing on joining until I read your
webpage. I'm hoping you can shed more light on this for me. Am I making
a big mistake here or is this a good business to try? Also, I still have
not gotten any good answers as to why the center cannot be opened more
than twice a week....
ME: Kumon is probably a "good business" for most of the 1280 franchise
owners in the U.S. I suppose it was a "mistake" for most of the owners
who shut down or sold their center to another instructor. I have no
idea what the figures are for the failures. If you pursue Kumon you
will get a book called a "UFOC" (Uniform Franchise Offering Circular).
Part of what hooked me in was a list of over a 1000 thriving centers,
and a list of just a few dozen that shut down. What a fantastic success
rate! If they can do it, why can't I??? However, much later I realized
the comparison was misleading; the list of operating centers was
complete, but the list of shutdowns was for just the previous year.
Cheat!
It's safe to say that if you want to work with students, Kumon is a
mistake. It's a self-study program. To have enough students to pay the
rent, you would hardly have time to work with students, anyway.
If you want the feeling of being in charge of your own business, Kumon
is probably a mistake. You will be very much under Kumon's control.
If you want to feel like you're making big improvements in the students'
academic abilities, I'm not sure what to say. I don't think there are
any studies that show Kumon to be beneficial, although I'll be the first
to admit such studies are very difficult to carry out.
It's not so much "why" the center can't be opened more than twice a
week, it's just that the Kumon routine is designed around two center
visits per week. Understand that the students don't do anything
differently at the center than they do at home. Kumon would work just
about as well with one visit per week, or as a total correspondence
course, even. I've always wondered if those two center visits per week
aren't mostly to make it look like Kumon gives you a lot of service for
your money!
***
I'm glad to find your site regarding G-B-U-Kumon. My son is ADHD and
has always struggled with math. He was always in small group math in
school until the "No Child Left Behind Laws" came into being. School
math has been a nightmare. My son's eighth grade teacher was
worthless - I begged for help through the school and it was impossible.
I found a tutor at $40 an hour that he could relate to and learn from
but it didn't help with his class. (My tutor has since moved.) We had
to attend summer school to retake the test to move up to 9th grade. At
my wits end, a home school mom told me about Kumon. So I talked to
Kumon and they assured me ADHD was not a problem and he could learn long
division. He was evaluated and found to be way behind and it would take
a year to catch him up to grade level. I decided to try it since he had
always struggled. He started out very very very basic (1+2=3, etc.)
finally 2 1/2 months ago we reached division. He mastered the basic 1
and 2 digit division problems rather quickly and finally moved up to
long division. For 2 months we have been doing the same work. The
instructor would give him a long division sheet and then say "oh, you're
struggling and I don't want you to be frustrated so go back and practice
8 divided by 2." Finally my son's frustration level exploded on me. He
was never allowed to show his work on the worksheet. He had to do
everything in his head and write the answer down. He also said they
didn't work with him and expected him to be able to look at the problem
and do it with no explanation. He said mom it's pure torture.
I just stopped taking him. I can't work long division problems out
without showing my work and I don't struggle with math. The one good
thing about Kumon, he did learn his times tables with ALL the repetitive
work with basic multiplication problems.
Now I'm looking for another way to help him. It's hard to find someone
that really has a knack for being able to help struggling kids learn.
Your website just confirmed I did the right thing to stop taking him to
Kumon.
ME: Thanks for sharing your experience. It points up the basic fallacy
with Kumon's, or anybody's, "self-study" method of instruction. If
self-study truly worked, we could take any 5-year-old who has started to
read and stick a mountain of books and worksheets in front of him and
say, "Ok, we'll be back in 15 years, at which time you'll be an
accomplished engineer, or accountant, or musician, etc." Everybody,
even the very best students, need help to get past a stumbling block.
It might be just a word or two to clear up a simple confusion, but
nothing substitutes for some words of experience from someone who's
already been down that road. (Not that everybody with experience has
good skills for passing it on.)
Most infuriating for me is that your son's instructor wouldn't let him
show the steps to the long division problems. If it makes something
doable that otherwise is very painful, then why not? It's an artificial
constraint; I can imagine many students who write the intermediate steps
beating, in both time and accuracy, the kids who struggle mightily to
hold everything in their heads. I know some Kumon instructors are
flexible enough to let students write the intermediate steps in long
division. I suspect most Kumon instructors are "by the book", though.
If I were working with your son, I would double check that he's rock
solid on the single-digit multiplications, and then try to make it clear
that division is nothing new, just multiplication turned around. After
all, if you're dividing 592 by 8, the first step is really asking
yourself, "Hmmm, now what's that number in the 8's table that's up in
the 50s? Oh, yeah, I remember; it's 56, which is . . . oh yeah, 8x7."
Whether you're a 5th-grader or a master mathematician, that's what's
going on in the brain. After getting the 56, some people might be able
to hang onto it, others might let it slip while they're writing down the
7. Then they have to bring it back by redoing the 7x8 multiplication.
I can't think of a reason on earth why he shouldn't be allowed to write
the 56 down so he can then see the upcoming subtraction easily.
If your son really has reached that stage, even with writing the steps
down, I'd say he's ready for division by multiple-digit divisors - where
*everybody* writes the intermediate steps down. (So what was the big
deal about *not* writing them down with single-digit divisors?)
If you find another tutor, I suggest that you retain that aspect of
Kumon whereby your son does 15 minutes or so of work every day.
Learning takes a combination of the two - direction from a "master", and
"doing".
***
THEE: I read your experience with the Kumon system. In your Q&A
section, a parent asked about Kumon workbooks. I found Kumon reading
and math workbooks in a school book store called ClassWorks in Hammond,
Louisiana. I purchased books for ages 5-7. Although I have not
enrolled my child in the Kumon program, I found the books to be
beneficial and "fun" to work on and my child completed the books quickly
and with interest. Just thought you might like to know.

ME: Thanks for writing, and the report on Kumon workbooks. I still have
never seen one. The people who run the educational book store in Dover,
Delaware, have never heard of Kumon. Glad to hear that the workbooks
are beneficial and fun - hard to imagine much higher praise than that!
***
THEE: I've been a high school English teacher for 24 years and my 13 year
old finds himself needing help with his basic math confidence and
skills. Kumon came from the school coundelor as a suggestion. I
googled it and read you before anything else and bless you for your
sensitive and candid insights. I don't need to know anything else.
ME: Thanks for visiting my Kumon page. I do worry that some
parents may reject Kumon based solely on my page. If your
child still can't do arithmetic, Kumon might help. Even though
Kumon could be much, much better for basic arithmetic,
it's better than nothing - a claim schools would be
hard-pressed to make with a straight face. If you went into
Kumon with your eyes open, and took my advice of doing everything
the student does, there's nothing to fear. You'll know how
bearable or not it is, and whether your child is benefiting.
You can stop any time. I'd love to hear back from parents
who have "done" Kumon.
***
THEE: I am an instructor in H~~, since 1999, and have shared your
frustration with the mind boggling insistance on following ineffective
and outdated operating procedures from the HO and other shortcoming of
the Kumon Program. However, I am of the opinion that Kumon is one of the
best answers for the middle income families struggling with the
ineffective schools in America. So I am trying to work within the system
to deliver a higher quality program to my students by delivering a
personalized lessons (bend but don't break).
This approach has worked well and have now grown to over 200 students.
However, I have reached a point where my personal time does not enable
me to continue to deliver the same level of service as the center
continues to grow.
Kumon in their wisdom, has opened 4 new center around me, one within 3
miles recently. My point of differentiation is the level of service and
I would like to develop my assistants, mostly high school students, to
continue this strategy: happy students + good grades = happy parents.
ME: Thanks a million for looking at my Kumon page and the kind words. I
had thought that H~~ was in the Washington, D.C. branch. If that's
correct, I'm a little surprised you haven't already brought down their
wrath for
>delivering personalized lessons (bend but don't break).
I will argue to the end that nothing I did could even count as
"bending"; I was just pulling students through the Kumon curriculum as
fast as they could handle it.
Regarding "helping students learn the objectives of the Kumon
worksheets", I didn't make much effort in my web page to hide the fact
that I think there are serious problems with what Kumon thinks is
important above the arithmetic levels. I think Kumon needs to tear it
all down and start from scratch. I think Kumon's goal should be to give
students a very solid, but very basic, foundation in each subject -
geometry, trigonometry, probability, calculus, etc. - so that if and
when they take it up in school or college, they can hit the ground
running and be in a good position to keep up when the material gets more
advanced.
***
THEE: I read attentively your comments on Kumon and they are ALL accurate.
There is NO place in the Kumon method for any meaningful instruction
(and certainly your intellect, writing skills and obviously top notch
teaching skills are of no interest to them and it is a true loss to
Kumon). It is a business and their sole interest is that each center
operate at 200 students. In order to make it viable they hire "very
qualified" assistants (high-school kids). This will make you laugh:
One of these qualified tutors corrected a D level page marking all
answers correct that indicated continuing division and not remainder
(example 5/2= 2.1 instead of 2R1).
Another point that you correctly state is that the mathematical
concept is NEVER approached. Students memorize and never learn a
mathematical strategy.
The reading program is an atrocity. I have observed kids that are
perfectly capable of "figuring" out a reading exercise (AI and AII)
proceeding by elimination (fill out the blanks) and NEVER actually
reading the content.
The "topping" is the Jr Kumon program. No 4-5 year old can concentrate
for 30 minutes on a particular task (physiologically the brain does
not acquire that capacity until 7-8 years old).
***
THEE: I was about to consider Kumon for my six-year old smart
daughter, but after reading your article online---I reconsidered.
I did observe my cousin's son when he was younger, doing Kumon under
time pressure and it was more of a torture than learning!
I went on to attend Columbia University without Kumon, so my daughter
should do well as well! Thanks again. You saved me lots of money and
headaches on this one.
ME: You're more than welcome. Of course, I try not to come out
and say, "Don't do Kumon!", because there are so many factors and
everyone is different. Coincidentally there was a very
entertaining article written recently by a woman who put herself
through half a year of Kumon.
http://www.slate.com/id/2152480
She didn't write the article from the angle, "Wow, if it's this
miserable for an *adult*, just think what it's like for a kid",
which is too bad. It would have been a real eye-opener.
***
THEE: May be you should have learned more math before buying a
franchise....if a student did not know a problem...I would show
him the steps! Such as how to solve a binomial or quadratic.
Kumon does help....the practice builds confidence, the memory of
basics help the student focus on the parts they need to as they
are half way thru the equation or problem.
It is better if a Kumon instructor is a math teacher or a math
major, they understand how to help the student. People such as
you should get a Curvers franchise or a food franchise and leave
the education to educators.
ME: But it was helping students that got me in trouble with
Kumon.
THEE: Well you need to get it off GOOGLE. Most parent are
ignorant and it hurts my business. For some reason as soon as
they search for KUmon the article appears...and like everything
there is more to the story.
The average American parents of the generation I want to sell
KUMON to, is not very up on education and does not get it...math
or reading....so doing KUMON is good especially if I can get them
to stay 6-months and read or get a kid to level J alg.
***
THEE: Thank you so much for your honest input regarding Kumon
Math. I have been suspicious of their methods for quite some
time. I had considered enrolling my son, but had kept hearing
that it required doing a series of worksheets for each level that
would have to be mastered before going on to the next. I do not
consider this method of teaching very inspiring, in fact, I think
it is down right mind numbing and only makes kids hate math.
However, it does sound very typical of Japanese methods and
methods in the United States around the 1950's. I think we will
reconsider Everyday Math through the University of Chicago.
Thanks again for your honesty.
ME: Thanks for spending time with my Kumon page. I do worry a
little that someone might reject Kumon solely on what I say, when
it might be very effective for that particular child. Good luck
with Everyday Math. I'm not familiar with it and will look into
it.
***
THEE: I stumbled upon your web page about Kumon. May I ask what
is the expected earning potential with owning your own center? I
have just recently started inquiring about Kumon.
ME: Figure the average center has 100 students. I think a
typical tuition now is about $115 per month. Kumon gets about
$35 of that. So you get to keep 100 x $80 = $8000 per month.
Out of that comes the rental of the center space and utilities.
That varies widely, of course, but I think very few Kumon
instructors get away with less than $2000 per month. You will
also need to pay about 4 assistants about $9 per hour for about 6
hours per week, each. Call that another $1000 per month (very
roughly), and you net about $5000 per month. Fair enough for
maybe 10 or 12 hours of work per week?
***
THEE: I just read your paper on Kumon (Kumon - the good, the bad,
& the ugly) after I found it on a google search. I am job
searching and came across a position with Kumon as an Education
Consultant. I'm very glad I read your take on the company, being
I am a math teacher and I cringed at the thought of rote learning
- isn't that what we strive to avoid in education these days?
Anyway, it almost immediately turned me away from applying for
the position because I could picture myself getting the same
emails about accusation of "changing the Kumon method." But then
it got me thinking... maybe this position is brand new. Did your
words turn the light switch on for Kumon? Are they looking to
improve their strategies in an effort to finally keep up with the
trends of American education?
ME: Interesting question - no one's ever asked me that! My gut
feeling is that my page has made Kumon powers think about their
program. I have no idea whether they might actually consider
making improvements, or just go with the status quo. I know they
never accepted my offer of cheap input!
There have been complaints in the Kumon instructors' forum about
certain math "sets" (a booklet of 10 worksheets) that are wildly
inappropriate where they're found in the curriculum. I heard
that Kumon management said they would look into it. I have no
idea if such discussions were prodded by my page to any extent.
I don't believe my page has a negative effect on Kumon's business
anymore than their own web site has a positive effect. Such a
business depends almost completely on word of mouth referrals.
People might visit Kumon's site after having heard about it from
another source, and they might even not reject it based on the
typically annoying hype they find there, but no one has ever
stumbled on Kumon's site cold and gone on to sign up. That's my
belief, anyway.
***
THEE: I very much enjoyed reading your thoughtful analysis of
Kumon. We have an 8 year old who has been in Kumon for over 2
years. I have had some of the same concerns you expressed even
though I also noted some benefits he gained from his experience.
My son will be away for one year and I am looking for a book from
which I could select some math problem for him on a regular
basis. His mother will be working with him on his reading. He is
going to the 4th grade. Any recommendation you may have will be
most appreciated.
ME: Unfortunately, it's a virtual impossibility comparing all the
hundreds or thousands of math programs out there and choosing
a favorite. I now tutor privately, and the funny thing is, I
don't care how good or bad a program is. If there are flaws, I
can turn that into something interesting and valuable working
with a student. "See, you know better than the people who wrote
this!" I tell the parents it doesn't matter to me - dollar store
workbooks, cd-rom programs, web-based programs, worksheets from
the internet... Having said that, it sounds like your son is
very advanced. You might consider the Math League Contest books,
which start at 4th grade. They are the exact opposite of Kumon -
fun; rich; words in every problem. Just search on "math league".
Hope that helps.
[Actually, I see the Math League material as being really only
beneficial when done under the wing of an enthusiastic, competent
tutor. Otherwise, it's just "either you know it or you don't"
review material, not learning material. With a good tutor, it can
be a fantastic tool getting a young mind to "think math".]
***
THEE: I feel you did an ok job of explaining the basis of kumon
(i.e. the worksheets and breakdown). However, I have worked for
kumon for 4 years and feel that there is not "bad" parts to
kumon. It all depends on the center and the child. Kumon is great
for students who are capable of learning on their own but not all
students can do this. Some students do need step by step
instructions. Also, at the center i work at I find that i work
with kids seperatly if they are really struggling. There is the
number chart, number board, number tracing, and as a teacher you
can explain doubles, and little tricks to do different math
problems. The method is the basis for the program but is the
instructors that make the biggest difference.
ME: Thanks for your thoughts. I suspect from what you write that
you haven't gotten involved in Kumon's math levels above basic
arithmetic. They're killers. I think when you refer to
"students who are capable of learning on their own", you really
mean those students who are fairly strong and who are still in a
review mode in Kumon's lower levels. I agree that "the
instructors make the biggest difference", but working with
students is what got me in trouble with Kumon.
***
THEE: Its sad to hear that you got in trouble for working with
the kids. The owner of the kumon I work at does have us use
guided questioning first but when that does not work she does let
us explain each step the the student even on the upper levels
such as H, I and J. I guess a lot has to do with the person that
runs the center.
ME: I ran my center.
***
THEE: Your website was invaluable in giving me an overall
understanding of the Kumon program. I have a 4 1/2 year old
starting to learn her letters and phonics and was considering
supplementing her reading development with the program. I've
decided against Kumon, primarily because of the emphasis on
timing.
She loves worksheets, though, and thrives on the few little pre-K
books I've picked up at Barnes and Noble and through our school
supply stores. Would you recommend using Kumon workbooks
outside of the program?
Thank you so much for your time and the candor of your site.
ME: Thanks for visiting, and thanks for writing. It might sound
funny, but I've never seen the Kumon workbooks sold in stores.
To be honest, you might want to reconsider the Kumon reading
program for your child. I didn't get into Kumon's beginning
reading levels on my page because it's not what people mainly
associate Kumon with; and my page was already too long.
I had a few very young students in the reading program, and the
results were pretty remarkable. One was not quite 4 years old.
Don't worry about the time limits at that stage. If there are
completion times on those worksheets (I forget) they would be
very "soft". No need to think about them at all.
Kumon's reading program for beginners may be the very best thing
Kumon has to offer. My own very personal reservation is the age-
old, is it good and is it necessary to start that young? And
such children are the ones sentenced to a lifetime of corrected
vision. But people always tell me I'm wrong about that.
***
THEE: Thanks for the heads-up on Kumon. They recently opened 2
centers near my home in AZ. With the poor educational system
here in the Phoenix area I was so close to enrolling my kids in
ANYTHING I thought might help pick up where the school is
dropping the ball. I guess I am going to stick with doing it
myself.
ME: To be honest, my main purpose in writing the page was not to
scare parents away from Kumon on my say-so. More than anything,
I would like parents to take me up on my suggestion that they do
everything the student must do in Kumon, and make a decision
based on that. I would love to hear back from parents who have
"done" Kumon.

***
THEE: I happen to come across your article just before I was to
make a decision to see a Kumon center being offered for resale.
In fact, I saw your article the day after I agreed to go and see
the site. I haven't seen it as yet because of your article.
I was wondering though weren't you aware of Kumon's method before
you undertook to investing in the franchise? I am a retired
teaher and, of course, my first impulse is to employ all methods
of instruction to facilitate a student's learning so in that
respect we both agree and I want to congratulate you. I
understand also that a prospective franchisee must complete all
worksheets to be familiar with the work that the student will
encounter. Did you do this? If so, didn't you see that the work
was repetitive and especially at the beginning covering many,
many worksheets? In other words, didn't you detect that the
system was flawed as you intimated in that respect? I must
confess that I am a Junior Kumon Instructor dealing with the
younger students in math and reading. I agree that the sheets
are repetitive and most of the students hate it. As you know, my
level must know the short sounds of the alphabet and with no
coaxing from my employers I initiated combining the sounds as
they learn them to formulate a word thus giving them a sense of
mastery and accomplishment. I suppose this is unKumon. I don't
know because I haven't been told otherwise. In fact, the
students do not even know why they are studying the alphabet
sounds unless I tell them which I always do.
I work for a highly successful Kumon franchise and thought that
I, too, could be successful with my educational background.
However, your article demonstrates that Kumon is a business first
with education a secondary goal for those (parents) with the
temperament, perseverance and stictuitiveness to succeed. This
method probably appeals to a certain segment of the population
with education as a major emphasis.
ME:
>I was wondering though weren't you aware of Kumon's method
before you undertook to investing in the franchise?
I learned about Kumon's method during training. Note that it's
not the Kumon method that I have a problem with. I think my page
makes it clear that it is the learning material itself. When
they first showed us some Kumon worksheets a couple of days into
training, I'll admit my heart sank at the barrenness of it. I
proceeded anyway because I was into it too far; I knew of no
other options at that time; and since I'd be there, it wouldn't
matter how bad the study material is. Which is what I tell the
parents of the students I tutor. I can actually turn weaknesses
and errors in the study material into a great learning experience
for the student.
>I understand also that a prospective franchisee must complete
all worksheets to be familiar with the work that the student will
encounter. Did you do this?
I think I more or less said in my web page that in the almost 3
years I was involved with Kumon, I did more Kumon worksheets than
anyone on the face of the earth, bar none.
>If so, didn't you see that the work was repetitive and
especially at the beginning covering many, many worksheets?
Repetitiveness, per se, is not a problem. I think in my web page
my only slight objection to repetition was in the long division
section. And a problem only arises there if the instructor is
hard-nosed about meeting the SCT.
>In other words, didn't you detect that the system was flawed as
you intimated in that respect?
The basic system is not flawed; it's great. Public schools
should adopt it.
>I must confess that I am a Junior Kumon Instructor dealing with
the younger students in math and reading. I agree that the
sheets are repetitive and most of the students hate it.
That's interesting. I had several very young students; one was
3. In my experience, they enjoyed Kumon the most.
Hope that helps in your decision-making process.
***
THEE: I read the kumon article/experiences on your geocities
website. I am very interested in working/teaching kids (interest
developed as I spent time with my kids understanding what they
are taught, how etc) and was thinking about starting a
afterschool learning center - looked at Kumon and also was
looking to start my own. I was wondering if you had any
tips/advice based on your own experience. Would appreciate any
help/tips from you.
ME: I'd be glad to share some thoughts. I'd rather you didn't
view them as tips or advice, though, just "for what they're
worth".
One rude awakening was that even though there is a kind of
pretense that a franchisee is an independent business owner, the
reality is that he is under the thumb of the franchiser to a much
greater extent than with any conventional employer. Put more
directly, Kumon treats you like a baby.
Working with or teaching kids is not part of the Kumon method.
How much you may be able to get away with that is unknown. It's
certainly not possible with a center large enough to generate an
average sort of income. I had in mind to "pull" students through
Kumon, but I knew from the beginning that would mean a small
center and poverty-level income.
I now have a private tutoring business. I live in Dover, the
capital of Delaware. It is the seat of state, county and city
government. I thought there would be parents willing to pay for
tutoring their kids. It looks as though there is not a resident
of Dover willing to pay $20 per half hour. If I had as much
business as I could handle at that rate, I would barely make a 5-
figure income. Even with Kumon, there was not a single parent
who ever expressed what a bargain $80 per month is. And that at
a time when the *average* tuition in the Southeast was $115.
I've heard of a center charging $150. [Dec 2007 note: The figure
$125 per month has been showing up in articles, such as in Time
magazine, as the average Kumon tuition.]
Schools here have always viewed me as the enemy, whether with
Kumon or in my private business.
If my web page was too large to digest, the most condensed
summary is: Kumon math is barren at all levels, and of doubtful
merit and maliciously hard at the higher levels.
Hope none of that scares you off. Everybody can't have the luck
I have!
***
THEE: AMEN!
I'm a 19 year old college student at Texas A&M. I just found
your site about Kumon and really wanted to thank you for what you
said. I was an instructor at a Kumon in my hometown over the
summer and COMPLETELY disagreed with much of what they were doing
there. I loved that they were trying to give kids that "sky is
the limit" goal, but I hated that there was only one way to do
something...and it was the "Kumon way".
I'm a science major and I know a thing or two about math. I know
about twenty different ways to do just about everything in math
and I've been told I'm a good teacher because of it. But when I
went to Kumon, I was ripped apart because of it. The head Kumon
instructor would CONSTANTLY talk to me about not teaching kids
the "right" way to do something.
For instance, I happened to be grading a girl's homework while
she was there doing her daily work. I was marking EVERY one
wrong. I asked her if she maybe wanted me to help explain to her
what was going on so she wouldn't have to keep doing sheets over
and over again until perfection was reached. She gratefully
accepted the help. I taught her the way I'd always added,
subtracted, multiplied, and divided fractions. I even showed just
a few little tricks that always helped me. By the end, she seemed
SO much happier. I graded her daily work and it had only one
wrong on it. She told me she felt so much better.
Then when she took the work up to the instructor to check it, he
promptly told her not to come to me for help anymore because I
was showing her the "wrong way"... AKA the "slow way". I never
quite understood how she was wrong for doing it that way when she
understood where the answers came from and got all the right
answers to boot.
I really hated being a part of something that had no interest in
understanding; just in time, time, time and perfection. I would
NEVER put a child through the Hell of Kumon.
Thanks for making me not feel so alone!
ME: Thanks for taking the time to write. It's also therapeutic
from my end to hear that I'm not totally crazy. Sounds like that
particular Kumon instructor should be banished from any kind of
work in education. Keep up the great work!
***
THEE: We were considering Kumon for my daughter and I found your
comments when I googled "Kumon" and now I'm not so sure. Do you
have any suggestions, based on your experience, regarding
materials that I might use at home to help my 3rd grade daughter
with math? I found a website that sells Otter Creek materials (a
suggestion from Aunt Patty's website) -just wondering if you knew
of any others.
ME: Is your daughter already behind in math? She's at the right
age where Kumon may actually be very beneficial in getting her to
about grade level. Then you can decide if you want to continue
with it. The irony about Kumon is that, as barren as the math
curriculum is, at least there's a chance a child will do it. You
could buy better math material anywhere, blindfolded, but how
many children will sit down and do daily assignments based on
parents' orders? On the other hand, "Kumon" is like some big,
old, omnipresent genie in the sky that has to be *obeyed*. Of
course, a lot of kids don't even fall for that in this day and
age and stuff their worksheets up above the ceiling tiles.
In summary, if you were considering Kumon, give it a try for a
couple of months. And, as I suggested in my web page, do
everything your daughter has to do. Then you will *know*.
Hope that helps.
***
THEE: A quick question. Is it possible to get one set of
all Kumon worksheets? Thanks
ME: As far as I know, that's not possible until you've
become a Kumon instructor. In case I'm wrong about that, you
should ask the Franchise Recruitment Manager for your area.
***
THEE: Hi, I read your comments about Kumon. I am
considering signing up my daughter to go there. Do you think
Kumon is beneficial for dislexic children or is it a waste of
time?. I am sure you had students with dilexia. what was their
experience like? My daughter is in 3rd grade with a reading level
of beginning of 2nd grade. She seems to be doing ok in math.
Would you recommend that I sign her up for math also or is it
"torture".
ME: I only had one student who had a dyslexia-like
condition where he didn't see words right. Sometimes I felt sure
I was seeing improvement, other times I was doubtful. Measuring
the "benefits" of any educational program is very difficult, and
there may be non-academic benefits which aren't obvious. Sorry
to be so noncommittal.
As far as putting a student into two Kumon classes, I would
really like to hear from parents who have followed my advice to
do everything the student has to do, and tell me whether they
think it's too much for the child.
THEE: Thank you. I am sure there are many types of
dyslexia and every child learns in various, individual ways. I
may try Kumon for six months and see my daughter's progress and
re-evalute everything then. Thanks for your time.
***
THEE: Thanks for your candid experience on Kumon. I can't
say I've read every single word on the BAD part but I do agree
with the gist of your feedback. My 3rd grader started Kumon last
November. Through Kumon's assessment, my child was only a year
behind in reading and math. I was told that in 8 months, he would
be caught up. Unfortunately, 10 months later, my child is still
repeating worksheets he has completed a few months ago. I had
taken for granted what was taught and had believed that Kumon
must be the best thing since sliced bread. It seems that a lot
of parents had great experiences with their child's progress. It
was just a month or so ago when I had taken the time to review
his work to realize that he's repeating the same stuff over and
over again. I finally woke up to the reality that my kid will
never move on because he will never be PERFECT in that level.
That's just my child!!! He just doesn't have the smarts for it.
They will not let my child move on by simply understanding how to
solve a problem; he will have to excel within the time limit
given on the test. Since he did not do well when timed, it's
pretty much hopeless on my end to believe that he can advance to
the next level. I'm disappointed at my experience. Now that my
child has started 4th grade, he's still (according to their
evaluation) on 2nd grade reading and 3rd grade math. As far as
I'm concerned, it just doesn't work for every single child. I
just had him pulled out of Kumon recently. I had spent enough
money and didn't feel that it was worth it anymore.
ME: Sorry about your unhappy Kumon experience. It sounds
like your Kumon instructor operated too much "by the book". My
impression is that most instructors recognize the
unreasonableness of the mastery requirements and will bend when
they see that a student can't meet them - which is almost all
students after the first few levels.
***
THEE: i just moved here [Newport richey florida] fr the
Philippines last February. i just finished reading your article
about kumon and it was very interesting for me since I have been
wondering whether to put my child who is a 2nd grader, into kumon
to help her with school stuff. I was wondering if you have any
suggestion as to where I can put my child beside kumon. Thanks
so much and have a good day.
ME: I really can't make a suggestion because there are so many
options out there that it's impossible to know them all. For
that matter, it takes years to get familiar with just one! Also,
it would depend on your child's particular situation.
If you were considering Kumon, I would rather you didn't reject
it based on my web page. At your child's age, it may be a very
good thing, particularly, for burning in basic arithmetic. If
you accept my advice to do what your child has to do, you will
know if and when it's time to drop Kumon.
***
THEE: I found your comments on Kumon when looking for the
nearest center, hoping that they would take my son, who is 23!
He needs to be strong in Math in order to enter College (after
getting first his GED). He happened to say yes to my suggestion
and I was thrilled about it.....
Nevertheless, your comments did not surprise me at all. When he
was a little boy, back in my country (Colombia), I was about to
register him. I guess I didn't make all the effort - distance,
price, and so forth, because there was back in my mind all those
things you said about the program. I follow my
intuition...consciously and some times unconsciously.
I mainly thought it could be good for him now, since he complains
that I did not teach/gave him discipline habits. I raised him
under the Summer Hill's philosophy. And I don't regret it.
Any suggestions about his Math obstacle? He is an avid reader,
and he writes beautiful poems, and he can talk very smart about
most any thing.
ME: I have heard of older students who made good progress
up through Kumon's middle levels. I think you know already that
it's not going to be a "quick fix"; that there's a *lot* of work
to be done. If your son is highly motivated, and the instructor
will allow him to do much greater workloads than the typical,
younger student, Kumon may give him a good jumping off point for
tackling the GED. Best of luck.
***
THEE: I READ YOUR INFORMATION ON KUMONS, AND I WOULD LIKE
TO GET MORE INFO FROM YOU IF POSSIBLE. I HOMESCHOOL MY BOYS AND I
ALSO HAVE THEM IN KUMONS FOR BOTH SUBJECTS.
ONES A 4TH GRADER AND THE OTHER A 6TH GRADER.. I HAVE BEEN GOING
TO THE KUMONS IN CALIFORNIA FOR ALMOST TWO YEARS. I AM JUST
WONDERING IF THIS MAY CAUSE MORE DAMAGE THAN GOOD. WHEN I
CONSIDER THE PRICE (380) A MONTH, I WONDER IF ITS ALMOST BETTER
TO HIRE A TEACHER OR COLLEGE STUDENT TO HELP, WHERE THE HELP IS
NEEDED.
THIS WAY ITS ONE ON ONE. THE INSTRUCTOR IS REAL NICE AND SMART,
BUT SHE CANT GET AROUND TO EVERYONE, AND I FEEL THAT THE BOYS
JUST SIT AND WORK IN A TABLE WITH A HIGH SCHOOL STUDENT THAT
KEEPS SAYING TO THEM, KEEP GUESSING TILL YOU GET IT RIGHT. THIS
INSTRUCTOR DOES NOT WANT THE MOMS TO CHECK ANYTHING AT HOME, SHE
SAYS ITS BETTER FOR THEM TO CATCH THE MISTAKES. PLEASE ADVISE ME
ON SOME MORE INFO.
ME: It's a very difficult thing to say what's "best" for
students. There are more options out there than anyone can get
familiar with. In your case, I would repeat what I say on my web
page: do the Kumon assignments before your children so you can
gauge how beneficial they are. For example, have your children
reached a point where they are just "spinning wheels"? I think
you can easily find a better math curriculum than Kumon's, but
can you or the tutor get your children to work through it? As
far as reading is concerned, I suspect the children's version of
Time magazine (Time for Kids) and National Geographic would be a
lot more interesting and valuable than the Kumon reading
worksheets.
***
THEE: subject Looking for a tutoring service
With great interest I read your paper and as a parent, I very
much appreciated your blunt honesty.
We have a 4.5 year old daughter in public school and though the
government has mandated smaller class sizes in Kindegarten
(1:20), I see that the teacher simply does not have time for one-
on-one interaction with her students. That said, I would like to
look into a reading (and perhaps math) tutoring program in order
to assist my daughter in getting ahead in school.
The three main programs that I have been looking into are Sylvan,
Oxford and Kumon. Most important to me is that my daughter see
this tutoring program as FUN. I don't want her to be at a desk
being talked at (she has years of schooling ahead of her in which
will be taught in that manner).
Can you perhaps suggest a service which you know to be a good
one, while it also respects the young age of my daughter and can
teach with that in mind?
P.S. It may be important to note that we live in Ontario Canada.
ME: This may come as a surprise, but if you think your daughter
should be in a reading program, I suggest you give Kumon's a
chance. I've seen remarkable results with some very young
students. I think Kumon's program for beginning readers may be
the best thing it has to offer. I didn't discuss it in my web
page because I don't think it's what Kumon is mainly associated
with. I think your daughter will find the worksheets fun.
THEE: Thank you for your surprising answer.
I will look into Kumon. Do you feel that the classes for very
young children are fun as opposed to more of a classroom-type
setting?
ME: For the very young children, there are two slightly
different possibilities. If your center runs the formalized
Junior Kumon program, your daughter will sit at a table with a
small number of other students. My own feeling is that, while
we're supposed to believe in the miraculousness of a small
student to teacher ratio, one teacher can't really accomplish
much with a small group of 3, 4, or 5, say, working on different
assignments. But just because something's not great doesn't mean
it's bad. I think the attention would be more enjoyable than a
classroom situation for most students. (I was the exception who
would rather "hide" in a big class than be exposed in a small
group.)
And we're only talking "center" days. A student does Kumon every
day, but only goes into the center twice or once a week. You
will be the one working with your child on a daily basis. That's
not a scam - you're paying for Kumon's program.
So the other possibility is a center that has not implemented the
formal Junior Kumon program, but accepts young students. In that
case, you will work with your child at the center on center days
just as you do at home.
***
THEE: subject Kumon alternative
I just read (most) of your critique on Kumon and am glad that I
did. Though I am not gullible enough to make my decisions based
on random opinions on the internet, I felt that there was enough
passion in your prose to convince me, or at least make me
reconsider, my thoughts of sending my child to Kumon. We have not
committed to any extra curricular help programs but I feel that
the time has come.
I have a seventh grader who is a "gamer". He struggles in school
but never gives up. His two older brothers seem to him to be able
to get along in the school environment with ease. I know this
makes him feel inferior and that he wants to work to be like them
in this regard, but our study sessions are getting more and more
contentious. We are both losing patience with each other.
Obviously, I don't want him to give up his fight. I feel that he
is getting more reluctant to ask me for help, yet I feel he still
needs it. This is a kid who can do mental math much better than I
can (very intuitive with numbers), but can't organize the steps
to reduce an equation to get an unknown by itself on one side of
the equal sign.
It seems to me that you have a deeper knowledge of these extra
help organizations, and certainly hands on experience with at
least one of them. Are you aware of any programs that you feel
may help us in any way? I will of course do my own due-diligence
and will not take your recommendations at face value. You seem
like a person committed to education, and any insights would be
appreciated.
ME: While I really don't want anyone to reject Kumon based
on a reading of my web page, it really sounds like Kumon is not a
fit for your son. If you said he was "nowhere" with math, I
might've suggested a few months of Kumon to review basic
arithmetic. But that's not the problem, and, in any case, it's
*very* difficult for any student in the middle grades to get up
to grade level in Kumon - not that there's much correspondence
between Kumon levels and grades above 5 or 6.
I'm fascinated by the specific problem you mention - not being
able to whittle away all the chaff to leave the unknown all by
itself on one side of the equation. I've probably heard myself
say, "Whatever you do to one side of the equation, you do to the
other," a million times by now. If a student can't "get"
something that simple and straightforward, it would have to
indicate a weakness in more basic material. In this case, I
would say the direct predecessor is evaluating involved
expressions, what Kumon calls "4-operation problems". Those
problems make the order of operations second nature. Then, when
you get to solving equations, you find yourself naturally peeling
everything away from the variable in the inverse order - like
pulling thugs off a friend from the outside in. When you say,
"organizing the steps", I would say they organize themselves; the
equation calls out what to do next. Unfortunately, you can't go
to Kumon and say, "I want to start with the 4-operation
problems."
Besides tracking down a workbook of 4-operation problems, the
only other suggestion, or question, that comes to mind is, do you
think he would work better with someone outside the family,
meaning a tutor? Might that eliminate the contentiousness?
***
THEE: Thanks for your insightful opinion regarding the
Kumon Franchise program. I have my 11 year old daughter now a
year in half involved in the Kumon program presently in E level.
She hates doing the monotonous and tediousness of the sheets. I
emphatically persist the benefit of repetitive math to improved
skill level.
I was raised on a method similar to Kumon prior to entering the
US educational system. The paper used and problems given by Kumon
reminded me of my Latin schools which incorporated very similar
rigorous methods long before Kumon arrived on the shores of the
USA. My husband himself raised on the exact methods used to fall
asleep in the American 9th grade Algebra classes due to low level
of education given to American high schoolers in contrast to his
classes obtained in a public school from his country of origin
which would have taught this math in 7th Grade equal to 9th
Algebra I.
It disappoints me to think you offer no other options in doing
nothing instead of something. The only barrier faced by an
immigrant child entering the USA would be language rather than
numbers since my earlier experiences have taught me well. My
initial deficiencies in education primarily dealt with language
writing & communication. Math was my panacea since obviously math
is a universal language. It saved me from failing grammar school
until I adopted the US level of math which was low in comparison.
My 9th grade Algebra I teacher never taught that year since
became ill; therefore, half my Algebra I was computer science to
my own detriment it effected my future in math until my
graduation - since I never recovered from not learning the
basics. To my own regret, it prevented me from pursuing a degree
in science due to lack of confidence in mathematics.
Ironically, I excelled in everything in school with the exception
of Math which had been my savior in middle school against the
English and reading classes. Would you know, I obtained a degree
in English. I entered the military and was unfortunately told my
talents were in math and science even more humorous.
I told my daughter who wants to become an mechanical engineer to
focus on math since it is arduous and incredibly puzzling, but
worth all the efforts due to benefits of the challenge in
obtaining the solutions to every day problems in science and
everything around us. In fact, it is the true language of man no
other language will succumb to its ability to supersede
boundaries of any language
ME: Thanks for your thoughts on math education. I think I
am being fair to Kumon in my web page. I feel that it could be
much, much better than it is. I think Kumon makes it very
difficult for even the best students to progress through its
levels. I agree that the material taught in Level E is
important. Is it making your daughter miserable because she
doesn't have a good grasp of it yet? Or can she keep her pencil
moving through those problems and get the right answers, but the
instructor repeats her because the time isn't "good enough"? I'm
guessing the former, and one of the shames of Kumon's "self-
study" approach is that a teacher or tutor may be able to explain
things in a few sentences that make everything clear. Before
adding and subtracting fractions, a student has to be a whiz at
breaking numbers into prime factors, and I don't think Kumon does
enough in that regard.
Regarding your husband sleeping in Algebra class, I have heard
that story a hundred times and I will never be sympathetic to it.
That should be an opportunity for a student to shine above all
his classmates. What a wasted opportunity. I hope in the work
world he doesn't sleep his way through a job just because it's
too "easy".
As far as offering other options to Kumon, I'm not qualified to
do that. It took me a couple of years just to get fully familiar
with Kumon, and there are surely hundreds of math programs out
there.
***
THEE: Hi, I was in the process of checking out Kumon
center opportunities near my home in rural west central Florida
when I saw your listing on my google search page. Interesting
info, particularly for someone (me) who is about ready to go to
Chicago for training. I was intially looking to purchase an
existing center but have been having 2nd thoughts for a number of
reasons - some of which were brought up in your discussion. I
would be interested in speaking with you more abour your
experience! Thanks!
ME: I'd be happy to try to answer any questions that I
can. If you meant jumping right in with a phone call, I think
I'd be more comfortable with the first round being a list of
questions by email. If it's too much to say in email, then we
could talk.
***
THEE: Is there a supplemental math program that you would
recommend over Kumon?
ME: I don't know Sylvan or Huntington - they're as secretive as
Kumon and I suppose you have to join up to see what you're
getting. I'd imagine almost any math curriculum would have to be
better than Kumon's. But whereas your child would not likely do
20 minutes of extra math a day on your say-so, he might do Kumon,
for a while, at least, since Kumon has the aura of something that
is "big" and "official", and costs money.
***
THEE: Thanks so much for the candid insider's perspective
of Kumon. I wish you lived in the Atlanta area so I could hire
you to tutor my kids in Math. I'll probably start them w/ Kumon
for extra practice and mainly to keep them off of the computer
games, which is a considerable lure to kids these days. If you
have any other suggestions, let me know. Sylvan Learning Center
seems too commercial, and is quite expensive. I'm open to hear
any other thoughts you may have.
ME: Your plan sounds good to me. Just keep close tabs on
the Kumon so you'll have a good idea when it's "done it's job."
I was in an educational book store just today and saw lots of
math workbooks that would make nice complements to Kumon - not
that your kids want even more! But they might be something to
try after Kumon is done. Some that looked pretty good to me, at
a quick glance, were "Daily Math Practice" (Evan Moore), "Daily
Math Warm-ups" (Carson Dellosa), "Real-World Math" (word
problems), "Word Problems" (Kelley Wingate), and "Jump Into
Math". Good luck!
P.S. About keeping kids off of computer games, what about
softball or kickball?
***
THEE: I've just read your information about Kumon. I now
have tears in my eyes. This was my plan to help my child. The
Kumon center is the most affordable of the supplemental centers
in our area. After reading what you wrote, I don't know where to
turn. I just want to help him. He is 8 in the 4th grade and
struggling. Do you have any suggestions? A very distraught mom.
ME: If Kumon is nowhere near as good as it could be, that
doesn't mean it is necessarily bad or valueless. Students in
mid-elementary school are probably the best candidates for Kumon.
At those levels, Kumon is pure arithmetic. That's only 40% of
what's taught in elementary schools nowadays, but it's the most
important thing. Obviously, your son needs to be drilled in
basic arithmetic, and that's what Kumon will do, even if it could
do it much better. Nobody else out there does it at all. Give
it a try; keep very close tabs on what your son is going through;
and don't give any thought now about the higher levels.
Don't worry!
P.S. Isn't 8 mighty young for the 4th grade?
***
THEE: I have just visted your site concerning Kumon. I am
desperately looking for something to help my boys get through
school. My oldest is 8 and is the kind of kid who thinks outside
the box. Which makes school tricky. I saw an ad about Kumon and
started to search it when I ran across your web page and I am
wondering if you have found a program you like better. I really
would just like something to supplement at home with. Do you
have any suggestions?
ME: That's a good question, and almost impossible for me
to answer. There are probably hundreds, or thousands, of
programs, and it took me a year or so to really get to know
Kumon. If your son is motivated and will do work on his own or
on your say-so, I would just suggest going to the nearest
educational resources book store and pick out some work books
that look reasonably fun. If you're saying your son thinks *too*
far outside the box, *too* much of the time, perhaps Kumon is
just what the doctor ordered for getting him to do a little
thinking *inside* the box. I would argue we need both skills.
At his age, he'd be looking at a couple of years of plus, minus,
multiply and divide, buy you could pull him out before it kills
him.
***
THEE: Here is a question from a Kumon Parent. My son just
started Kumon reading and has been completing the 2a1 level
within less than 1 minute to 1 minute window for each sheet with
a 100 percent accuracy. His teacher assigned him the same work
sheets over again to work on for the whole week which I found
baffling. And I asked why, her answer was that Kumon suggests
repititions. But I thought Kumon encouraged a student to go at
his own pace. Should I consider moving him to another center that
would allow him to move at his pace? I hope this e-mail finds
you.
ME: I'm really not in a position to say whether the reading
repetitions are necessary or unnecessary. In fairness,
repetition is a fundamental component of Kumon, and by signing
up, one is showing one's acceptance of that. I would recommend
against moving to another center in the blind hope that the pace
will be "better" there. That's an unknown, and the instructor
certainly can't make any guarantees upfront. I will say that, as
a Kumon instructor, the parents who demanded that their children
just blast through everything, one time, made things *very*
difficult. Kumon is "slow and steady wins the race." My own
belief is that *everything* benefits by being read twice. You
always see something new the second time around. Knowing how a
story, for example, ends makes clearer things that lead up to the
ending. All this is to say that I always assigned the reading
worksheets twice, even if a student did a so-called "perfect" job
the first time. But my technique was to send the student all the
way through the level, and then all the way through again. That
way he isn't rereading something he just read a few days ago,
making it somewhat fresher. If that sounds more agreeable to
you, you might ask the instructor if she would go for that plan.
***
THEE: I read every word of your Kumon pieces (G/B/&theU),
as my wife is hot-to-trot on us purchasing a Kumon franchise.
I've been reluctant from the beginning, as it has just "seemed"
to me that it's a whole truck-load of work (for an educated
ageing boomer like me) for the amount of salary the center-
owner/instructor would be able to pay him/herself from the
average sized center. Of course, we couldn't get any specifics
out of the local center we interviewed (in San Antonio); and you
know what the Kumon company "line" is on anticipated profits.
I was wondering if you'd be kind enough (and willing) to give me
the "low down" on center ownership profitablity. When I, myself,
put the numbers to it (at $100/student/month) it seems like one
would need a proverbial truck-load of students to pay oneself the
kind of money we need to keep our lifestyle afloat (>$120K/yr.).
I'd be grateful for whatever light you'd be willing to shed on
Kumon profitability (for serious lookers like us).
ME: It looks to me like you've crunched the numbers
correctly. I've never even *thought* what it would take to net
$120K/yr - probably two large centers! It sounds to me like
you'd have to have 100 students at $135/mo. - and a rent-free
business space.
If it's hard to get rich in Kumon, keep in mind it's only about 2
days of work per week after you find your groove. I went into
Kumon just looking to net in the mid to high teens.
***
THEE: Thank you so much for your comments about Kumon. I
am thinking of putting my 4th-grade son in a program that will
help him develop focus, and I think Kumon may do that, but so
might other programs that are more supportive. I will be
investigating Huntington and other options (some locally based).
Glad you are encouraging parents to do the worksheets to see what
their children are being asked to do.
ME: Thanks for the kind words. So far, I've gotten almost
no feedback from parents who have taken my suggestion to do the
Kumon assignments, which disappoints me somewhat.

Contact Donald Sauter: send an email; view guestbook; sign guestbook. Back to Donald Sauter's main page. Back to the top of this page.
And if you liked this one, please visit my page of Scrabble thoughts! 
geovisit();